Re: [algogeeks] Re: help me debug this

2011-01-17 Thread juver++
Redirect your output to the file, and you'll see that at the end of line you have extra blank. You need to write something like this (in all sections): for(i=j;i<(j+2*C-1);i++) { if (i != j) printf(" "); printf("%d",s[i]); // note there is no space } -- You received this message because you ar

Re: [algogeeks] Re: help me debug this

2011-01-17 Thread ankit sablok
i improved upon my code but still i get a presentation error dunno wts the judge judging it shows me the correct way when i output test cases on my compiler but on the judge it says wrong answer or presentation error #include #include #include #include #include using namespace std; int prime(int

[algogeeks] Re: Bit Manipulation

2011-01-17 Thread anurag.singh
Yes, Following should do for next smallest: 1. Find rightmost 01 pair 2. swap these two bits (make it 10) e.g. N=3(011), Next smallest: 5(101) N=10(1010), Next smallest: 12(1100) N=14(01110), Next Smallest: 22(10110) On Jan 18, 12:48 am, juver++ wrote: > @abobe > my solution is wrong when number

[algogeeks] Re: Bit Manipulation

2011-01-17 Thread anurag.singh
Yes, I guess following correction in step 1 for Next Smallest should fix it: 1. Find the leftmost 1 [ Not rightmost]. On Jan 18, 12:48 am, juver++ wrote: > @abobe > my solution is wrong when number is even (but it can be avoided with some > corrections into an implementation), > btw, you have a m

[algogeeks] Re: Bit Manipulation

2011-01-17 Thread anurag.singh
Yes, I guess following correction in step 1 for Next Smallest should fix it: 1. Find the leftmost 1 [ Not rightmost]. On Jan 18, 12:48 am, juver++ wrote: > @abobe > my solution is wrong when number is even (but it can be avoided with some > corrections into an implementation), > btw, you have a m

[algogeeks] Re: Bit Manipulation

2011-01-17 Thread juver++
@abobe my solution is wrong when number is even (but it can be avoided with some corrections into an implementation), btw, you have a mistake: N=3(011), Next smallest: 6(110) ,* Should be 101 (5)!* In other cases my version is correct. -- You received this message because you are subscribed t

[algogeeks] Re: Bit Manipulation

2011-01-17 Thread anurag.singh
Q1 is very well answered by Sunny but looks like Q2 is still open. IMHO, juver++ soln on Q2 doesn't look correct to me (OR I didn't understand the problem). if given no is N=2 (010), then next smallest no N+1 = 3 (011) is not the right answer as it has TWO 1's. I guess, for N=2 (010), next smallest

[algogeeks] Re: help me debug this

2011-01-17 Thread juver++
Got AC with your code with small corrections to the output - don't use getchar(); output specification says: Each line of output should be followed by a blank line (so, add blank line to match the sample output) you print a whitespace after each number, so the last character in your line is a w

[algogeeks] help me debug this

2011-01-17 Thread ankit sablok
i dont knw wt wrong i have done in this simple problem bt its nt being accepted at uva judge here is the link to the problem http://uva.onlinejudge.org/index.php?option=com_onlinejudge&Itemid=8&category=6&page=show_problem&problem=347 please help me debug my code #include #include #include #inc

[algogeeks] Re: Amazon Interview - Algorithms

2011-01-17 Thread bittu
here is Working Code Without DP in which we need To Find Out Minimum Jumps & for every Jumps its Finding Maximum Step That Can Be Cover In A Jump So that No. of Jumps Required is Minimized.. #include int main() { int arr[]={1 ,3, 5 ,8, 9, 2, 6 ,7 ,6, 8, 9}; int size=sizeof(a)/sizeof(int); int

Re: [algogeeks] Re: Bit Manipulation

2011-01-17 Thread juver++
@Sunny Good! -- You received this message because you are subscribed to the Google Groups "Algorithm Geeks" group. To post to this group, send email to algogeeks@googlegroups.com. To unsubscribe from this group, send email to algogeeks+unsubscr...@googlegroups.com. For more options, visit this

Re: [algogeeks] Re: Bit Manipulation

2011-01-17 Thread sunny agrawal
Q1: initially compute xor of all the values from 0 to n in a variable Temp so temp = 0^1^2^n let result is used to store the missing number for each ith bit of missing number where i = 0-31 we can find it as following ith bit of result = (xor of all ith bits of values of array) xored with (

Re: [algogeeks] Re: google paper/...plz help..its urgent

2011-01-17 Thread pacific pacific
thanks very much. On Sun, Jan 16, 2011 at 5:04 PM, Lakhan Arya wrote: > @pacific > > Sets of size 2 can have 2 elements common with set of size greater > than 2. for example if set is (1,2) than it is adjacent to sets like > (1,2,3) (1,2,4), (1,2,3,4...n) etc. > So (1,2) is adjacent to (1,2,3),

[algogeeks] Re: Bit Manipulation

2011-01-17 Thread awesomeandroid
@juver i am really sorry ,i forget to mention.ya this soln will work only if n is even power of 2. Regards Priyaranjan http://code-forum.blogspot.com On Jan 17, 12:43 pm, juver++ wrote: > @awesomeandroid > Your solution for Q1 is wrong. It can be applied only for such numbers N = > 2^k, so numb

Re: [algogeeks] Re: Bit Manipulation

2011-01-17 Thread juver++
@above, no :) it is solvable in linear time. -- You received this message because you are subscribed to the Google Groups "Algorithm Geeks" group. To post to this group, send email to algogeeks@googlegroups.com. To unsubscribe from this group, send email to algogeeks+unsubscr...@googlegroups.co

Re: [algogeeks] Re: Bit Manipulation

2011-01-17 Thread Sharath Channahalli
I think Q1 is NP hard problem since the number of bits grows exponentially as the array size increases. On Mon, Jan 17, 2011 at 1:13 PM, juver++ wrote: > @awesomeandroid > Your solution for Q1 is wrong. It can be applied only for such numbers N = > 2^k, so number should power of 2. > > -- > You